matheraum.de
Raum für Mathematik
Offene Informations- und Nachhilfegemeinschaft

Für Schüler, Studenten, Lehrer, Mathematik-Interessierte.
Hallo Gast!einloggen | registrieren ]
Startseite · Forum · Wissen · Kurse · Mitglieder · Team · Impressum
Forenbaum
^ Forenbaum
Status Schulmathe
  Status Primarstufe
  Status Mathe Klassen 5-7
  Status Mathe Klassen 8-10
  Status Oberstufenmathe
    Status Schul-Analysis
    Status Lin. Algebra/Vektor
    Status Stochastik
    Status Abivorbereitung
  Status Mathe-Wettbewerbe
    Status Bundeswettb. Mathe
    Status Deutsche MO
    Status Internationale MO
    Status MO andere Länder
    Status Känguru
  Status Sonstiges

Gezeigt werden alle Foren bis zur Tiefe 2

Navigation
 Startseite...
 Neuerdings beta neu
 Forum...
 vorwissen...
 vorkurse...
 Werkzeuge...
 Nachhilfevermittlung beta...
 Online-Spiele beta
 Suchen
 Verein...
 Impressum
Das Projekt
Server und Internetanbindung werden durch Spenden finanziert.
Organisiert wird das Projekt von unserem Koordinatorenteam.
Hunderte Mitglieder helfen ehrenamtlich in unseren moderierten Foren.
Anbieter der Seite ist der gemeinnützige Verein "Vorhilfe.de e.V.".
Partnerseiten
Weitere Fächer:

Open Source FunktionenplotterFunkyPlot: Kostenloser und quelloffener Funktionenplotter für Linux und andere Betriebssysteme
StartseiteMatheForenUni-Komplexe AnalysisSingularitäten
Foren für weitere Schulfächer findest Du auf www.vorhilfe.de z.B. Geschichte • Erdkunde • Sozialwissenschaften • Politik/Wirtschaft
Forum "Uni-Komplexe Analysis" - Singularitäten
Singularitäten < komplex < Analysis < Hochschule < Mathe < Vorhilfe
Ansicht: [ geschachtelt ] | ^ Forum "Uni-Komplexe Analysis"  | ^^ Alle Foren  | ^ Forenbaum  | Materialien

Singularitäten: Frage (beantwortet)
Status: (Frage) beantwortet Status 
Datum: 20:12 So 06.07.2014
Autor: Differential

Aufgabe
$0$ sei ein Pol in $f$, d.h. [mm] $$|f(z)|\to\infty\;\;\;\text{für }z\to [/mm] 0$$ Dann ist $0$ eine hebbare Singularität von [mm] $$g(z):=\frac{zf'(z)}{f(z)}$$ [/mm]

Das $0$ ein Pol ist sagt uns [mm] $$|g(z)|=|zf'(z)|\frac{1}{|f(z)|}\to 0\;\;\;\text{für }z\to [/mm] 0$$ Doch was können wir daraus weiter folgern? Hier komme ich einfach nicht weiter.

Liebe Grüße
Differential

        
Bezug
Singularitäten: Antwort
Status: (Antwort) fertig Status 
Datum: 20:49 So 06.07.2014
Autor: felixf

Moin!

> $0$ sei ein Pol in $f$, d.h.
> [mm]|f(z)|\to\infty\;\;\;\text{für }z\to 0[/mm] Dann ist $0$ eine
> hebbare Singularität von [mm]g(z):=\frac{zf'(z)}{f(z)}[/mm]
>
>  Das $0$ ein Pol ist sagt uns
> [mm]|g(z)|=|zf'(z)|\frac{1}{|f(z)|}\to 0\;\;\;\text{für }z\to 0[/mm]

Nein, das stimmt nicht.

> Doch was können wir daraus weiter folgern? Hier komme ich
> einfach nicht weiter.

Wenn das stimmen wuerde, koenntest du den []Riemannschen Hebbarkeitssatz anwenden.

Alternativ kannst du aber auch wie folgt vorgehen: habe $f$ in 0 einen Pol der Ordnung $n$. Dann kannst du $f(z) = [mm] z^{-n} \cdot [/mm] g(z)$ (fuer $z [mm] \neq [/mm] 0$) schreiben mit $g(z)$ um 0 herum holomorph und dort ohne Nullstelle. Damit kannst du jezt $z f'(z) / f(z)$ etwas genauer anschauen.

LG Felix


Bezug
                
Bezug
Singularitäten: Frage (beantwortet)
Status: (Frage) beantwortet Status 
Datum: 21:43 So 06.07.2014
Autor: Differential

Hallo Felix,

ich bezeichne dein $g$ mal mit $h$, da $g$ schon in der Eingangsfrage verwendet wurde. Mit den neuen Überlegungen erhalten wir [mm] $$g(z)=z\frac{h'(z)}{h(z)}-n$$ [/mm] Damit ist mir aber noch nicht viel geholfen, denn nun sieht $g$ fast genau so aus wie vorher. Lediglich eine holomorphe Funktion ($f$) wurde durch eine andere ($h$) ersetzt und von letzterer weiß ich jetzt sogar noch weniger ($f$ hatte einen Pol in $0$).

Oder können wir noch mehr sagen?

Liebe Grüße
Differential

Bezug
                        
Bezug
Singularitäten: Antwort
Status: (Antwort) fertig Status 
Datum: 22:34 So 06.07.2014
Autor: felixf

Moin!

> Hallo Felix,
>  
> ich bezeichne dein $g$ mal mit $h$, da $g$ schon in der
> Eingangsfrage verwendet wurde. Mit den neuen Überlegungen
> erhalten wir [mm]g(z)=z\frac{h'(z)}{h(z)}-n[/mm] Damit ist mir aber

Du meinst wohl eher [mm] $\cdot [/mm] (-n)$, als $n$ vom Term abziehen?

> noch nicht viel geholfen, denn nun sieht $g$ fast genau so
> aus wie vorher. Lediglich eine holomorphe Funktion ($f$)
> wurde durch eine andere ($h$) ersetzt und von letzterer
> weiß ich jetzt sogar noch weniger ($f$ hatte einen Pol in
> $0$).

Oh doch, du weisst viel mehr. $h$ hat naemlich in der Naehe von $0$ keine Nullstelle.

LG Felix


Bezug
                                
Bezug
Singularitäten: Frage (beantwortet)
Status: (Frage) beantwortet Status 
Datum: 00:08 Mo 07.07.2014
Autor: Differential

Hm, wir haben doch [mm] $$f'(z)=\frac{1}{z^n}\left\{h'(z)-n\frac{h(z)}{z}\right\}$$ [/mm] Damit folgt für mich [mm] $$\frac{zf'(z)}{f(z)}=\frac{zh'(z)-nh(z)}{h(z)}=z\frac{h'(z)}{h(z)}-n$$ [/mm] Wo ist der Fehler?

Das [mm] $h(z_0)\ne [/mm] 0$ gilt, ist klar. Aber was genau bringt mir das? Bitte gehe etwas mehr ins Detail.

Liebe Grüße
Differential

Bezug
                                        
Bezug
Singularitäten: Antwort
Status: (Antwort) fertig Status 
Datum: 06:12 Mo 07.07.2014
Autor: fred97


> Hm, wir haben doch
> [mm]f'(z)=\frac{1}{z^n}\left\{h'(z)-n\frac{h(z)}{z}\right\}[/mm]
> Damit folgt für mich
> [mm]\frac{zf'(z)}{f(z)}=\frac{zh'(z)-nh(z)}{h(z)=z\frac{h'(z)}{h(z)}-n[/mm]
> Wo ist der Fehler?
>  
> Das [mm]h(z_0)\ne 0[/mm] gilt, ist klar. Aber was genau bringt mir
> das? Bitte gehe etwas mehr ins Detail.


Zeige, dass der Grenzwert [mm] \limes_{z\rightarrow 0}\frac{zf'(z)}{f(z)} [/mm] in [mm] \IC [/mm] existiert.

FRED

>  
> Liebe Grüße
>  Differential


Bezug
                                                
Bezug
Singularitäten: Frage (beantwortet)
Status: (Frage) beantwortet Status 
Datum: 13:07 Mo 07.07.2014
Autor: Differential

Tut mir leid für die verhunzte Formel; habe das oben mal korrigiert. Du bist aber leider nicht darauf eingegangen, ob es nun richtig ist oder nicht, denn ich sehe da keinen Fehler.

Ich denke mal, dass du meinst, dass ich zeigen soll, dass der Grenzwert [mm] $$z\frac{h'(z)}{h(z)}$$ [/mm] existiert und damit $0$ kein Pol in dieser Funktion ist, oder?

Wäre dem nicht so, so wäre $0$ ein Pol zweiter Ordnung in $f$, d.h. [mm] $$f(z)=\frac{1}{z^2}\tilde{h}(z)$$ [/mm] Ist das der richtige Ansatz? Und wie führt das zu einem Widerspruch?

Liebe Grüße
Differential

Bezug
                                                        
Bezug
Singularitäten: Antwort
Status: (Antwort) fertig Status 
Datum: 13:37 Mo 07.07.2014
Autor: fred97

Es ist

  [mm] $\bruch{z*f'(z)}{f(z)}=\bruch{z*h'(z)}{h(z)}-n$ [/mm]

Das hattest Du schon richtig. Wegen h(0) [mm] \ne [/mm] 0 folgt:

[mm] \limes_{z\rightarrow 0} \bruch{z*f'(z)}{f(z)}=\bruch{0*h'(0)}{h(0)}-n=-n$ [/mm]

Der Riemannsche Hebbarkeitssatz sagt nun: [mm] $\bruch{z*f'(z)}{f(z)}$ [/mm] hat in 0 eine hebbare Singularität.

FRED

Bezug
                                                                
Bezug
Singularitäten: Frage (beantwortet)
Status: (Frage) beantwortet Status 
Datum: 15:12 Mo 07.07.2014
Autor: Differential

Den Satz kenne ich leider nicht, aber folgt nicht auch ganz einfach, dass in einer Umgebung von $0$ die Funktion beschränkt ist? Das würde ja genügen, um den Nachweis für die Hebbarkeit von $0$ erbracht zu haben.

Gruß
Differential

Bezug
                                                                        
Bezug
Singularitäten: Antwort
Status: (Antwort) fertig Status 
Datum: 19:58 Mo 07.07.2014
Autor: fred97


> Den Satz kenne ich leider nicht, aber folgt nicht auch ganz
> einfach, dass in einer Umgebung von [mm]0[/mm] die Funktion
> beschränkt ist? Das würde ja genügen, um den Nachweis
> für die Hebbarkeit von [mm]0[/mm] erbracht zu haben.

Das ist doch der erwähnte Satz !!!!

FRED

>  
> Gruß
>  Differential


Bezug
                                                        
Bezug
Singularitäten: Antwort
Status: (Antwort) fertig Status 
Datum: 06:35 Di 08.07.2014
Autor: fred97

Du kannst es auch so machen:  Die Funktion h ist auf einer Umgebung U von 0 holomorph und wegen h(0) [mm] \ne [/mm] 0 können wir annehmen, dass U so "klein" ist, dass gilt: h(z) [mm] \ne [/mm] 0 für alle z [mm] \in [/mm] U.

Für z [mm] \in [/mm] U [mm] \setminus \{0\} [/mm] gilt:

   (*) $ [mm] \bruch{z\cdot{}f'(z)}{f(z)}=\bruch{z\cdot{}h'(z)}{h(z)}-n [/mm] $

Die rechte Seite von (*) ist auf U holomorph. Damit hat  [mm] \bruch{z\cdot{}f'(z)}{f(z)} [/mm] eine holomorphe Fortsetzung auf U.

FRED

Bezug
                                                                
Bezug
Singularitäten: Mitteilung
Status: (Mitteilung) Reaktion unnötig Status 
Datum: 07:23 Di 08.07.2014
Autor: felixf

Moin Fred,

> Du kannst es auch so machen:  Die Funktion h ist auf einer
> Umgebung U von 0 holomorph und wegen h(0) [mm]\ne[/mm] 0 können wir
> annehmen, dass U so "klein" ist, dass gilt: h(z) [mm]\ne[/mm] 0 für
> alle z [mm]\in[/mm] U.
>  
> Für z [mm]\in[/mm] U [mm]\setminus \{0\}[/mm] gilt:
>  
> (*)
> [mm]\bruch{z\cdot{}f'(z)}{f(z)}=\bruch{z\cdot{}h'(z)}{h(z)}-n[/mm]
>  
> Die rechte Seite von (*) ist auf U holomorph. Damit hat  
> [mm]\bruch{z\cdot{}f'(z)}{f(z)}[/mm] eine holomorphe Fortsetzung auf
> U.

genau das schwebte mir vor :-)

LG Felix


Bezug
                                        
Bezug
Singularitäten: Mitteilung
Status: (Mitteilung) Reaktion unnötig Status 
Datum: 07:23 Di 08.07.2014
Autor: felixf

Moin!

> Wo ist der Fehler?

Sorry, da hatte ich mich verguckt...

LG Felix


Bezug
Ansicht: [ geschachtelt ] | ^ Forum "Uni-Komplexe Analysis"  | ^^ Alle Foren  | ^ Forenbaum  | Materialien


^ Seitenanfang ^
www.schulmatheforum.de
[ Startseite | Forum | Wissen | Kurse | Mitglieder | Team | Impressum ]